LSAT and Law School Admissions Forum

Get expert LSAT preparation and law school admissions advice from PowerScore Test Preparation.

 smile22
  • Posts: 135
  • Joined: Jan 05, 2014
|
#14572
I was able to narrow the answers down to A, C, and E. I eliminated A, because I felt it did not connect the rogue element in the stimulus ("should never provide psychotherapy on talk shows"). Is this why A is incorrect? I ran into trouble figuring out how to choose between answers C and E. I ended up choosing E; however, I am still confused as to why answer C is incorrect as both C and E appear to be stating similar things. Could you please explain why C is incorrect and E is correct?
 Nikki Siclunov
PowerScore Staff
  • PowerScore Staff
  • Posts: 1362
  • Joined: Aug 02, 2011
|
#14582
Hey smile22,

Thanks for your question! This is a fairly rare Assumption—PR question stem, not a Justify—PR stem, because it asks us to identify a principle that must be assumed in order for the conclusion to be properly drawn. Since “must” is a necessary condition indicator, the correct answer will be principle upon which the argument depends. By contrast, a Justify—PR question would have required us to identify a principle, which, if assumed, would enable the conclusion to be properly drawn, i.e. a principle sufficient to prove the conclusion.

The psychologist’s argument is structured as follows:
Premise:
  • Psychotherapists who attempt to provide psychotherapy on talk shows are expected to entertain a broad audience.
Premise:
  • Satisfying this demand is nearly always incompatible with providing high-quality psychological help.
Conclusion:
  • Psychotherapists should never provide psychotherapy on talk shows.
As a general rule, when prephrasing a principle to answer any Help Family question, look for logical gaps or deficiencies in the argument that need to be fixed. In this problem, the author argues that psychotherapy should never be provided on talk shows because such therapy is unlikely to be of high quality (“satisfying this demand is nearly always incompatible with providing high-quality psychological help”). This line of reasoning is certainly objectionable: even if talk shows fail to provide the ideal context for psychological help, one could argue that for some people substandard psychotherapy is better than none.

To identify a principle upon which the argument depends, we must establish that therapy should never be provided in a context in which it is unlikely to be of high quality. Answer choice (E) fits the bill. To double-check if this is an assumption, apply the Assumption Negation Technique—logically negate the answer and ask yourself if the following statement would undermine the argument:
Psychotherapists can sometimes attempt to provide psychological help even if the manner in which such help is provided makes it unlikely to be of high quality.
Since the logical opposite of this answer choice directly undermines the conclusion of the argument, answer choice (E) contains an assumption upon which the argument depends.

Now, let's take a look at answer choice (C). This is an attractive answer choice, especially if one misreads the question stem. Indeed, if we adopt the principle that psychotherapy should never be provided in a context in which there is any chance that the therapy might of less than high quality, the author would be justified in reaching her conclusion. After all, therapy on talk shows is “nearly always” of less than high quality, which would certainly meet the standard set forth in answer choice (C).

Unfortunately, this principle is not an assumption upon which the conclusion depends. It simply sets the bar for finding psychotherapy objectionable too low. To double-check if this is an assumption, apply the Assumption Negation Technique—logically negate the answer and ask yourself if the following statement would undermine the argument:
Psychotherapy can sometimes be provided in a context in which there is some chance that the therapy might be of less than high quality.
The author would reply by observing that talk shows pose a substantially higher chance that the therapy provided on them would be of less than high quality. Since the logical opposite of answer choice (C) does not weaken the conclusion of the argument, it is not an assumption upon which the argument depends.

Generally speaking, the correct answer to an Assumption question will not go too far: it must be a statement upon which the conclusion depends, not a statement that proves the conclusion. An assumption can be considered a “minimalist” answer, and cannot contain exaggerations or elements that are extraneous to the argument.

Hope this helps!
 nschlesi
  • Posts: 6
  • Joined: May 25, 2018
|
#49225
I am still having a lot of trouble distinguishing between answer choices C and E. :oops: It seems like they are saying the exact same thing. I also don't see how (C) is not an assumption, but rather a way to "set the bar for finding psychotherapy objectionable too low" to quote Nikki. I see that when negated, answer choice (E) weakens the argument, but negated (C) also seems to weaken the argument.
Thanks.
 deck1134
  • Posts: 160
  • Joined: Jun 11, 2018
|
#49666
I'm generally good at telling the difference between question types, but I don't see it here. How do we know that this is an assumption question? I assumed that it was a justify, which led me to (C), a perfect justify answer (which it is, right?)

So This is pretty scary, as if I make that mistake on test day, I'll be completely screwed.
 Adam Tyson
PowerScore Staff
  • PowerScore Staff
  • Posts: 5153
  • Joined: Apr 14, 2011
|
#49686
nschlesi, the key difference in those answers is in their strength. Answer C is incredibly strong - don't do it if there is ANY chance of it being low quality. To negate that, we might say "it's okay to do it in situations where there is some chance of it being low quality." That weakens the argument, perhaps a little, but doesn't destroy it, because the author wasn't arguing about there being SOME chance but about it being UNLIKELY to be of high quality. Our author might be okay with some risk, but not where a low quality outcome is probable.

Answer E, negated, would be that it's okay to do it sometimes when the risk is very high that the outcome will be of low quality. That directly attacks the argument.

Deck, the key in the stem is in the flow of information. The stem asks "which of the following MUST be assumed" (emphasis added.) A Justify stem will never ask which answer must be true, but will instead ask IF (key word) the answers are true, which one allows the conclusion to be properly drawn/follows logically/must be true. That is, in a Justify question, it's the answer that is taken as true and the conclusion that must be true when you add the right answer. In an assumption, the right answer must be true, at least in the mind of the author, but it need not prove the conclusion (and usually won't).

Let me point that out again - a Justify stem will use the word "if", as in "which of the following, IF true" or "which of the following, IF assumed." An assumption stem will NOT use the word "if" because there is no if about it - the correct answer MUST be something the author believed.

They love to mess with us in the language, as here where they added "in order for the psychologist’s conclusion to be properly drawn" or when they talk about "does most to justify" or "helps to justify" (both of which indicate Strengthen questions), or where they use "if assumed" and then you fall into the trap of thinking it is an Assumption question. Pay attention to the flow of information whether from the stimulus to the answer choices or from the answer choices back up to the stimulus, and that will help you avoid making those sorts of mistakes.
 deck1134
  • Posts: 160
  • Joined: Jun 11, 2018
|
#49882
Tricky Tricky.


Thanks, Adam! I'm usually pretty good with question types, but this one threw me for some reason!
 Leela
  • Posts: 63
  • Joined: Apr 13, 2019
|
#64664
Adam Tyson wrote:nschlesi, the key difference in those answers is in their strength. Answer C is incredibly strong - don't do it it there is ANY chance of it being low quality. To negate that, we might say "it's okay to do it in situations where there is some chance of it being low quality." That weakens the argument, perhaps a little, but doesn't destroy it, because the author wasn't arguing about there being SOME chance but about it being UNLIKELY to be of high quality. Our author might be okay with some risk, but not where a low quality outcome is probable.

Answer E, negated, would be that it's okay to do it sometimes when the risk very high that the outcome will be of low quality. That directly attacks the argument.

Deck, the key in the stem is in the flow of information. The stem asks "which of the following MUST be assumed" (emphasis added.) A Justify stem will never ask which answer must be true, but will instead ask IF (key word) the answers are true, which one allows the conclusion to be properly drawn/follows logically/must be true. That is, in a Justify question, it's the answer that is taken as true and the conclusion that must be true when you add the right answer. In an assumption, the right answer must be true, at least in the mind of the author, but it need not prove the conclusion (and usually won't).

Let me point that out again - a Justify stem will use the word "if", as in "which of the following, IF true" or "which of the following, IF assumed." An assumption stem will NOT use the word "if" because there is no if about it - the correct answer MUST be something the author believed.

They love to mess with us in the language, as here where they added "in order for the psychologist’s conclusion to be properly drawn" or when they talk about "does most to justify" or "helps to justify" (both of which indicate Strengthen questions), or where they use "if assumed" and then you fall into the trap of thinking it is an Assumption question. Pay attention to the flow of information whether from the stimulus to the answer choices or from the answer choices back up to the stimulus, and that will help you avoid making those sorts of mistakes.
Hi Adam,

I'm not sure I understand your explanation above. Could you please elaborate further or perhaps explain in a different way how answer choice C doesn't sufficiently weaken the stimulus when negated? Both C and E were appealing answer choices to me and both appeared to weaken the stimulus when negated.
 Adam Tyson
PowerScore Staff
  • PowerScore Staff
  • Posts: 5153
  • Joined: Apr 14, 2011
|
#64679
Sure thing, Leela! The standard for the Negation Technique is not that the negated answer has to weaken the argument. That standard is too low. Instead, the negated answer must completely destroy the argument, so that the conclusion does not follow from the evidence.

Now let's break down the argument with some simple paraphrasing. The conclusion is "never provide help on talk shows." The evidence is "that kind of help is almost never good." The correct answer will link those two ideas together, and our prephrase should be along the lines of "if it almost never helps, don't do it." Answer E does that perfectly, which is why it is the correct answer. Answer C, on the other hand, does not link the "almost never helps" idea. Instead, it links to a new idea - "it might not help."

The negation of C is something like "it's okay to provide that help if there is some chance it won't be good." Does that destroy the argument, that if it's almost never good you should not do it? Not at all! The author could easily say "Sure, you can do it is there is some chance it won't be good, but not in a situation where it is almost never good. A little risk of not being good is fine, but not a high risk."

Honestly, I am not sure that answer C, negated, even weakens the argument. When you negate E, however, the argument is completely destroyed. "It's okay to try to help even when it probably won't be very good." If that is true, then the premises do not support the conclusion at all. There's your standard, and your winner.

I hope that was help of high quality!
 Leela
  • Posts: 63
  • Joined: Apr 13, 2019
|
#64699
Thank you! That was very helpful. I'm realizing that I didn't accurately negate C, which is why it seemed appealing.
 LSAT student
  • Posts: 32
  • Joined: Aug 23, 2020
|
#78673
Hello,

I picked choice (C) over (E) because choice (C) uses the word "context", and choice E says "manner". But it seemed like there was greater emphasis on context in the stimulus. I know I got the answer wrong, but was I on the right track? Is there something else that makes C wrong that I'm not getting?

Get the most out of your LSAT Prep Plus subscription.

Analyze and track your performance with our Testing and Analytics Package.